cardiovascular

79
Welcome [email protected] | Sign out Home My Account GP ST Entry Latest News Testimonials About us GP ST Entry - Stage 2: Test History You scored: 55% Total time taken to complete the test: 971 minutes 1 seconds. Total number of questions answered: 64, out of: 64 possible. Average time taken per question: 15 minutes 10 seconds. Question Details: Extended Matching Question Maximum Mark: 3 Mark Awarded: 1 Time taken to answer: 5 minutes 44 seconds. Main Question For each patient below, choose the most appropriate cause of the murmur. Select ONE option only from the list. Each option may be selected once, more than once or not at all. Answers 1. 55 year old female presents with gradual onset SOB, palpitations & fatigue. Examination findings reveal a malar flush, AF and a rumbling mid-diastolic murmur loudest in expiration with the patient lying on the left side Your Answer: A. Aortic regurgitati on Correct

Upload: ns223vimal

Post on 24-Nov-2014

165 views

Category:

Documents


0 download

TRANSCRIPT

Page 1: cardiovascular

Welcome [email protected] | Sign out Home My Account GP ST Entry Latest News Testimonials About us

GP ST Entry - Stage 2: Test HistoryYou scored: 55%

Total time taken to complete the test: 971 minutes 1 seconds.Total number of questions answered: 64, out of: 64 possible.Average time taken per question: 15 minutes 10 seconds.Question Details:Extended Matching QuestionMaximum Mark:   3Mark Awarded:   1Time taken to answer:   5 minutes 44 seconds.Main QuestionFor each patient below, choose the most appropriate cause of the murmur.Select ONE option only from the list.Each option may be selected once, more than once or not at all.Answers

1. 55 year old female presents with gradual onset SOB, palpitations & fatigue. Examination findings reveal a malar flush, AF and a rumbling mid-diastolic murmur loudest in expiration with the patient lying on the left side

Your Answer:A. Aortic regurgitationCorrect Answer:D. Mitral stenosis

2. 67 year old male 4 days post myocardial infarction has increasing SOB. Bibasal lung crepitations and a pansystolic murmur can be heard on auscultation.

Your Answer:C. Mitral regurgitationCorrect Answer:Your answer is correct!

3. 18 month old female presents with failure to thrive and recurrent chest infections. Examination findings reveal a collapsing pulse, left subclavicular thrill and a “continuous machinery-like” murmur loudest at the left upper chest.

Your Answer:B. Aortic stenosisCorrect Answer:F. Patent ductus arteriosus

Additional Question InfoQ1 – D

Page 2: cardiovascular

Features of mitral stenosis include fatigue, dyspnoea, palpitations, malar flush, AF, tapping non-displaced apex beat, opening snap and rumbling mid-diastolic murmur (loudest in expiration and patient on left side).Q2 – C

Mitral regurgitation may result post MI from ischaemic damage to the papillary muscles. Patient presents with features of heart failure and mortality is significantly increased.Q3 – F

Patent ductus arteriosus occurs when the ductus arteriosus fails to close after birth. The typical child with a patent ductus arteriosus (PDA) is asymptomatic. Occasionally, a history of feeding difficulties and poor growth during infancy is found. However, frank symptoms of congestive heart failure are rare. As many as one third of children with patent ductus arteriosus are small for their age. In the presence of significant pulmonary overcirculation, tachypnea, tachycardia, and a widened pulse pressure may be found.The apical impulse is laterally displaced. A thrill may be present in the suprasternal notch or in the left infraclavicular region.

The classic patent ductus arteriosus murmur has been referred to as a machinery murmur, which is continuous loudest at the left upper chest. The peripheral pulses are often referred to as bounding. This is related to the high left ventricular stroke volume, which may cause systolic hypertension. The phenomenon of bounding pulses also is caused by the low diastolic pressure in the systemic circulation as blood runs off from the aorta into the pulmonary circulation. 

Send us your feedback about this question.

Extended Matching QuestionMaximum Mark:   3Mark Awarded:   1Time taken to answer:   3 minutes 37 seconds.Main QuestionFor each ECG change below, choose the most appropriate cause.Select ONE option only from the list.Each option may be selected once, more than once or not at all.Answers

1. Prolonged PR interval, depressed ST segment & inverted T wave.

Your Answer:F. Acute pericarditisCorrect Answer:D. Hypokalaemia

2. Prolonged QT interval Your Answer:D. HypokalaemiaCorrect Answer:

Page 3: cardiovascular

B. Hypocalcaemia

3. ST depression, inverted T wave in leads V5-6.

Your Answer:G. Digoxin toxicityCorrect Answer:Your answer is correct!

Additional Question InfoQ1 - D

Hypokalaemia can present with muscle weakness, hypotonia, arrhythmias, cramps & tetany. ECG show a prolonged PR interval, depressed ST segment, small or inverted T waves and a prominent U wave (after T wave).Q2 - B

Hypocalcaemia can present with tetany, perioral paraesthesiae, carpo-pedal spasm if brachial artery occluded (Trousseau’s sign) and neuromuscualar excitability (Chvostek’s sign is facial muscle twitching when VIIn is tapped). ECG shows a prolonged QT interval.Q3 - G

Digoxin toxicity can present with arrhythmias (ventricular ectopics & bradycardia), confusion, nausea, insomnia, agitation and xanthopsia (predominance of yellow in vision). ECG changes show ST depression and inverted T waves in V5-6 (reversed tick). 

Send us your feedback about this question.

Extended Matching QuestionMaximum Mark:   3Mark Awarded:   3Time taken to answer:   6 minutes 4 seconds.Main QuestionFor each patient below, choose the most appropriate cause.Select ONE option only from the list.Each option may be selected once, more than once or not at all.Answers

1. A 55 year old male presents with sudden severe constant central chest pain radiating to the neck unrelieved by GTN. On examination he is pale, sweaty, hypotensive and bradycardic. ECG shows ST elevation in the inferior leads.

Your Answer:E. Myocardial infarctionCorrect Answer:Your answer is correct!

2. A 40 year old male presents with sharp retrosternal chest pain worse on deep inspiration relieved by sitting forward. A “scratching”

Your Answer:B. Acute

Page 4: cardiovascular

sound is head on auscultation over the left lower sternal edge. ECG shows concave or saddles-shaped ST elevation in most of the leads.

pericarditisCorrect Answer:Your answer is correct!

3. A 65 year old male with advanced colorectal malignancy presents with chest pain, haemoptysis and shortness of breath. His saturations are 80% on air and ECG shows new onset AF and right bundle branch block.

Your Answer:A. Pulmonary embolismCorrect Answer:Your answer is correct!

Additional Question InfoQ1 - E

Myocardial infarction (MI) usually results from plaque rupture with thrombus formation in a coronary vessel. Diagnosis of acute MI requires 2 out of the following 3: cardiac type chest pain; evolutionary changes in serial ECGs; rise in serum cardiac markers. Although the clinical presentation of a patient is a key component in the overall evaluation of the patient with MI (classically sudden severe central chest pain radiating to the arms, neck or jaw, unrelieved by GTN often accompanied by sweating, nausea and shortness of breath), many events are either "silent" or are clinically unrecognized.

The appearance of cardiac markers in the circulation generally indicates myocardial necrosis and is a useful adjunct to diagnosis. Cardiac markers help to categorize MI, which is considered part of a spectrum referred to as acute coronary syndrome that includes ST-elevation MI (STEMI), non–ST-elevation MI (NSTEMI), and unstable angina. This categorization is valuable because patients with ischaemic discomfort may or may not have ST-segment elevations on their electrocardiogram. Those without ST elevations may ultimately be diagnosed with NSTEMI or with unstable angina based on the presence or absence of cardiac enzymes. Additionally, therapeutic decisions, such as administering an intravenous thrombolytic or performing percutaneous coronary intervention (PCI), are often made based on this categorization.Q2 - B

Acute pericarditis is inflammation of the pericardium characterized by chest pain, pericardial friction rub, and serial electrocardiographic changes. Classically precordial chest pain worse during inspiration, when lying flat, or during swallowing and with body motion, shortness of breath, may be pyrexial, tachycardic or tachypnoeic. A pericardial friction rub best heard over the over the left lower sternal edge is pathognomonic. ECG shows concave or saddle-shaped ST elevation in at least 2 limb leads and chest leads (most marked in V3-V6).Q3 - A

Pulmonary embolism (PE) is a blockage of the pulmonary artery or one of its branches, usually occurring from an embolized dislodged deep vein thrombus usually from the pelvis or legs. Venous stasis from long haul flights is a risk factor. Other risk factors include recent surgery, stroke or MI; disseminated malignancy; thrombophilia; pregnancy, Pill or HRT.

The classic triad of haemoptysis, dyspnoea and chest pain is rarely the case occurring in fewer than 20%

Page 5: cardiovascular

of patients. Signs may include cyanosis, tachypnoea, tachycardia, hypotension, raised JVP or pleural rub. ECG may show tachycardia, AF, RBBB, right ventricular strain, SIQIIITIII rare. ABG may show ↓PaO2 & ↓PaCO2. Do D-dimers only in those who do not have a high probability of PE as a negative result will exclude need for imaging. Positive result does not prove diagnosis and imaging required. CXR may show oligaemia or small pleural effusion. CTPA (or V/Q scan) is investigation of choice. Tx is at least 3mths of warfarin (LMWH until INR>2).

  

Send us your feedback about this question.

Single Best Answer QuestionMaximum Mark:   1Mark Awarded:   0Time taken to answer:   1 minutes 39 seconds.Main QuestionA 50 year old male presents with acute myocardial infarction. The following are contraindications to thrombolysis except:AnswersYour answer:C. Active cavitating lung disease

Correct Answer:D. AmyloidosisAdditional Question Info

Thrombolytic drugs are contra-indicated in recent haemorrhage, trauma, or surgery (including dental extraction), coagulation defects, bleeding diatheses, aortic dissection, aneurysm, coma, history of cerebrovascular disease especially recent events or with any residual disability, recent symptoms of possible peptic ulceration, heavy vaginal bleeding, severe hypertension, active pulmonary disease with cavitation, acute pancreatitis, pericarditis, bacterial endocarditis, severe liver disease, and oesophageal varices; also in the case of streptokinase, previous allergic reactions to either streptokinase or anistreplase (no longer available). 

Send us your feedback about this question.

Single Best Answer QuestionMaximum Mark:   1Mark Awarded:   1Time taken to answer:   0 minutes 58 seconds.Main QuestionAtypical presentation of MI is commoner in which of the following patients?AnswersYour answer:B. Elderly diabetic females

Correct Answer:Your answer is correct!Additional Question Info

Atypical presentation of MI is relatively common. Patients describe atypical pain attributing it to

Page 6: cardiovascular

indigestion. 30% of patients do not report any chest pain.

They may present with LVF, collapse, syncope, confusion, stroke. Patients tend to be elderly, female, history of diabetes or heart failure and mortality is higher. 

Send us your feedback about this question.

Extended Matching QuestionMaximum Mark:   3Mark Awarded:   3Time taken to answer:   3 minutes 1 seconds.Main QuestionFor each patient below, choose the most appropriate diagnosis. Select ONE option only from the list. Each option may be selected once, more than once or not at all.Answers

1. A 62 year old male with a history of angina presents with central chest pain at rest radiating to the left arm lasting half an hour which is relieved by GTN. ECG shows ST depression and Troponin levels are normal.

Your Answer:B. Unstable anginaCorrect Answer:Your answer is correct!

2. A 55 year old male presents with crushing central chest pain radiating to his jaw unrelieved by GTN. ECG shows ST elevation with left bundle branch block. Troponin levels are elevated.

Your Answer:D. Acute coronary syndrome with ST segment elevation (STEMI)Correct Answer:Your answer is correct!

3. A 60 year old female presents with precordial chest pain associated with nausea. ECG changes show ST depression & T wave inversion. Troponin is elevated.

Your Answer:E. Acute coronary syndrome without ST segment elevation (NSTEMI)Correct Answer:Your answer is correct!

Additional Question InfoQ1 - B

The term unstable angina was first used 3 decades ago as the intermediate state between myocardial infarction (MI) and stable angina. Unstable angina belongs to the continuum of the acute coronary syndromes which include those whose clinical presentations cover the following range of diagnoses: unstable angina, non–ST-elevation myocardial infarction (NSTEMI), and ST-elevation myocardial infarction (STEMI). Unstable angina is not associated with myocyte necrosis as evidenced by a lack of elevation in cardiac markers.

Page 7: cardiovascular

Q2 - D

Acute coronary syndrome with ST-segment elevation (STEMI) is what used to be known as an acute MI.

Q3 - E

In acute coronary syndrome without ST-segment elevation (NSTEMI), significant myocyte necrosis can occur without ST-elevation.

ECG may show ST depression, T-wave inversion or non-specific changes. Cardiac markers will be elevated. This group also includes the non-Q wave or subendocardial MI. 

Send us your feedback about this question.

Single Best Answer QuestionMaximum Mark:   1Mark Awarded:   1Time taken to answer:   1 minutes 36 seconds.Main QuestionTroponin T is a highly specific and sensitive marker for cardiac myonecrosis. How many hours after myocardial infarction is it maximally accurate?AnswersYour answer:C. 12

Correct Answer:Your answer is correct!Additional Question Info

Page 8: cardiovascular

The troponins are regulatory proteins found in skeletal and cardiac muscle. The 3 subunits that have been identified include troponin I (TnI), troponin T (TnT), and troponin C (TnC). They appear in serum within 4-8 hours after symptom onset (similar in timing to the release of CK-MB), are maximally accurate at 12 hours, however, they remain elevated for as long as 7-10 days post-MI.

One of the medicolegal pitfalls for the clinician is to mistakenly rule out NSTEMI on the basis of a single negative determination of troponin in the early 3- to 6-hour time frame after symptom onset. The troponins are recommended for evaluation of patients who present more than 24 hours after symptom onset. The cardiac troponins are sensitive, are cardiospecific, and provide prognostic information for patients with acute coronary syndrome (ACS). They have become the cardiac markers of choice for patients with ACS. 

Send us your feedback about this question.

Single Best Answer QuestionMaximum Mark:   1Mark Awarded:   1Time taken to answer:   0 minutes 47 seconds.Main QuestionThe following are ECG changes associated with myocardial infarction except?AnswersYour answer:A. Saddle shaped ST elevation

Correct Answer:Your answer is correct!Additional Question Info

Saddle-shaped ST elevation occurs in acute pericarditis.

Within hours of an MI the T wave may become peaked & ST segments may begin to rise.

Within 24 hours, the T wave inverts, as ST segment elevation begins to resolve. ST elevation rarely persists. T wave inversion may or may not persist.

Within a few days, pathological Q waves begin to form. Q waves usually persist but may resolve in 10%. 

Send us your feedback about this question.

Single Best Answer QuestionMaximum Mark:   1Mark Awarded:   0Time taken to answer:   6 minutes 32 seconds.Main QuestionFollowing a myocardial infarction which long-term drugs are recommended to reduce mortality?AnswersYour answer:C. Aspirin, Beta-blocker, ACE-inhibitor,

Page 9: cardiovascular

Correct Answer:D. Aspirin, Beta-blocker, ACE-inhibitor, StatinAdditional Question Info

Post myocardial infarction (MI) Aspirin, β-blocker, ACE-inhibitor, and Statin is recommended long term. Low dose aspirin (75-150mg) reduces vascular events (MI, stroke or vascular death) by 30%.

Long term β-blockade reduces mortality from all causes by 25% in all patients with previous MI. If contraindicated verapamil or diltiazem are alternatives.

ACE-inhibitors reduce mortality by up to 30% in those with heart failure. Reducing cholesterol has shown to be of benefit in patients with both elevated and normal cholesterol. 

Send us your feedback about this question.

Single Best Answer QuestionMaximum Mark:   1Mark Awarded:   0Time taken to answer:   0 minutes 33 seconds.Main QuestionAll the following are complications of MI except?AnswersYour answer:D. Pericarditis

Correct Answer:E. Mitral stenosisAdditional Question Info

Mitral regurgitation and not stenosis occurs due to rupture/ischaemia of the chordal or papillary muscles. Presents as pulmonary oedema. 

Send us your feedback about this question.

Single Best Answer QuestionMaximum Mark:   1Mark Awarded:   0Time taken to answer:   0 minutes 59 seconds.Main QuestionAll the following are causes of right axis deviation on an ECG except?AnswersYour answer:E. Wolff-Parkinson-White syndrome

Correct Answer:C. Inferior MIAdditional Question Info

Page 10: cardiovascular

Inferior MI causes left axis deviation. Anterolateral MI causes right axis deviation. 

Send us your feedback about this question.

Single Best Answer QuestionMaximum Mark:   1Mark Awarded:   0Time taken to answer:   6 minutes 27 seconds.Main QuestionAll the following may cause low voltage QRS complexes (<5mm in all limb leads) except:AnswersYour answer:E. Chronic obstructive pulmonary disease

Correct Answer:C. Pulmonary embolismAdditional Question Info

Diagnostic Criteria of Low voltage on the surface electrocardiogram may be either: 1. Voltage of entire QRS complex in all limb leads <5mm.2. Voltage of entire QRS complex in all precordial leads < 10mm.

It Is caused by an increase in the distance between the heart and the ECG leads, infiltration of the heart muscle itself and metabolic abnormalities are all associated with low voltage.

1. Increased Distance Pericardial effusionObesityCOPD with hyperinflationPleural effusionConstrictive pericarditis

2. Infiltrative Heart Disease AmyloidosisSclerodermaHaemachromatosis

3. Metabolic Abnormality Myxoedema

 

Send us your feedback about this question.

Single Best Answer QuestionMaximum Mark:   1Mark Awarded:   0Time taken to answer:   2 minutes 45 seconds.Main QuestionThe following are contraindications to exercise ECG testing except:AnswersYour answer:

Page 11: cardiovascular

B. Recent MI in the last 2 days

Correct Answer:C. Aortic regurgitationAdditional Question Info

Exercise testing is a cardiovascular stress test using treadmill bicycle exercise with ECG and blood pressure monitoring. Exercise stress testing, which is now widely available at a relatively low cost, is currently used most frequently to estimate prognosis and determine functional capacity, to assess the probability and extent of coronary disease, and to assess the effects of therapy.

Absolute contraindications: Acute myocardial infarction (within 2 d) Unstable angina not previously stabilized by medical therapy Uncontrolled cardiac arrhythmias causing symptoms or hemodynamic compromise Symptomatic severe aortic stenosis Uncontrolled symptomatic heart failure Acute pulmonary embolus or pulmonary infarction Acute myocarditis or pericarditis Acute aortic dissection

 

Send us your feedback about this question.

Single Best Answer QuestionMaximum Mark:   1Mark Awarded:   1Time taken to answer:   6 minutes 2 seconds.Main QuestionA 24 year old male intravenous drug user presents with a 3 day history of fatigue, chills, sweats, and progressively worsening shortness of breath. Examination findings reveal a high grade fever, painless macular lesions on his palms and soles and a new systolic murmur. A chest x-ray shows evidence of heart failure. What is the most likely diagnosis?AnswersYour answer:C. Acute bacterial infective endocarditis

Correct Answer:Your answer is correct!Additional Question Info

Infective endocarditis (IE) is an infection of the endocardial surface of the heart causing valvular insufficiency, which may result in congestive heart failure and myocardial abscesses.

Historically, infective endocarditis has been clinically divided into acute and subacute presentations. The diagnosis of subacute IE is suggested by a more indolent process. Acute IE is a much more aggressive disease with the potential for rapid onset congestive heart failure.

A history of antecedent invasive procedures or illicit drug use must be investigated. The classical diagnostic triad is persistent fever, emboli & new or changing murmurs. Other possible features include weight loss and anorexia; malaise; night sweats; clubbing after 1-2 months (now rarely seen); splenomegaly; anaemia (usually normocytic, rarely haemolytic); arthritis; embolic events – stroke/ recurrent pulmonary emboli in right sided endocarditis/ haematuria; mycotic aneurysm, (potentially causing subarachnoid haemorrhage or affecting the popliteal artery); nephritis (usually focal proliferative but may be diffuse); vasculitic - splinter haemorrhages & Janeway lesions (painless palmar

Page 12: cardiovascular

or plantar macules); immunologic phenomena - Osler's nodes (painful pulp infarcts in fingers or toes) & Roth's spots (boat shaped retinal haemorrhages with pale centre).

Splinter Haemorrhages:

 

Send us your feedback about this question.

Single Best Answer QuestionMaximum Mark:   1Mark Awarded:   0Time taken to answer:   3 minutes 59 seconds.Main QuestionAll the following are part of the Duke criteria for diagnosing infective endocarditis except:AnswersYour answer:C. Positive blood cultures in 2 separate cultures

Correct Answer:E. Weight lossAdditional Question Info

The diagnosis of endocarditis with the modified Duke criteria is based on pathologic or clinical findings.

Pathologic criteria for definite infectious endocarditis include microorganisms on cultures or histology in a vegetation or histologic confirmation of active disease in a vegetation or intracardiac abscess.

Clinical criteria for definite infectious endocarditis includes 2 major, 1 major and 3 minor, or 5 minor criteria, as follows:

Major criteria 1. Positive blood cultures (2 separate cultures for a typical endocarditis microorganism, such as

Streptococcus viridans or HACEK organism [Haemophilus parainfluenzae, Haemophilus aphrophilus, Haemophilus paraphrophilus, Actinobacillus actinomycetemcomitans, Cardiobacterium hominis, Eikenella corrodens, or Kingella species], persistently positive blood cultures, or evidence of infection with a Coxiella organism and/or Q fever)

2. Positive echocardiographic findings (eg, oscillating mass and/or vegetation, paravalvular abscess, or dehiscence of a prosthetic valve)

Page 13: cardiovascular

3. New valvular regurgitation

Minor criteria 1. Predisposition (history of IV drug use or congenital heart disease)2. Fever with a temperature of more than 38°C3. Vascular phenomena (arterial emboli, septic pulmonary infarcts, intracranial hemorrhage,

conjunctival hemorrhage, Janeway lesions [painless, hemorrhagic lesions on palms and soles])4. Immunologic phenomena (glomerulonephritis, Osler nodes, Roth spots, positive result for

rheumatoid factor)5. Positive blood culture findings without meeting the criteria above or serologic evidence of

active infection consistent with endocarditis

 

Send us your feedback about this question.

Extended Matching QuestionMaximum Mark:   3Mark Awarded:   1Time taken to answer:   11 minutes 17 seconds.Main QuestionFor each patient below, choose the most appropriate advice. Select ONE option only from the list. Each option may be selected once, more than once or not at all.Answers

1. A 55 year old male post ST elevation MI (STEMI) discharged from hospital yesterday wants to know when can he drive again?

Your Answer:D. 2 monthsCorrect Answer:C. 1 month

2. Same patient wants to know when he can resume sexual intercourse?

Your Answer:D. 2 monthsCorrect Answer:C. 1 month

3. He works as a postman. When do you advise he should go back to work?

Your Answer:D. 2 monthsCorrect Answer:Your answer is correct!

Additional Question InfoQ1 -C

Driving must cease for at least 1 week after: Angioplasty ST elevation myocardial infarction with successful angioplasty Pacemaker implantation

Driving must cease for at least 4 weeks after: Coronary Artery Bypass Graft ST elevation MI Non-ST elevation MI (not followed by successful angioplasty)

Driving should cease if: Patient has angina at rest (until symptoms are controlled) Left ventricular assist device is inserted

Page 14: cardiovascular

DVLA should be notified if: Patient has abdominal aortic aneurysm ≥6 cm diameter. NB: Patient disqualified if diameter >6.5 cm Arrythmias present (if patient incapacitated) Left ventricular assist device in situ

Q2 - C

Sexual intercourse is best avoided for one month.

Q3 -D

Most people can return to work in 2 months. A few occupations should not be restarted post MI such as airline pilot or airtraffic controllers. Drivers of public service or heavy goods vehicles my be permitted to return to work if they meet certain criteria. Patients with heavy manual labour should be advised to seek a lighter job. 

Send us your feedback about this question.

Single Best Answer QuestionMaximum Mark:   1Mark Awarded:   0Time taken to answer:   30 minutes 46 seconds.Main QuestionThe following are causes of atrial fibrillation except:AnswersYour answer:C. Hyperthyroidism

Correct Answer:A. Aortic valve diseaseAdditional Question Info

Atrial fibrillation (AF) is a supraventricular tachyarrhythmia characterized by disorganized chaotic atrial electrical activity and progressive deterioration of atrial electromechanical function. Electrocardiographic manifestations of atrial fibrillation include absence of P waves; rapid oscillations (or fibrillatory waves) that vary in amplitude, frequency, and shape; and an irregular ventricular response.

Common causes include: Myocardial infarction and myocardial ischaemia Mitral valve disease, especially mitral stenosis, for example, following rheumatic fever Hypertension Hyperthyroidism Alcohol - the most common cause in men less than 60 years old

Page 15: cardiovascular

Rarer causes include: Cardiomyopathy Lung carcinoma Atrial myxoma Constrictive pericarditis Endocarditis Haemochromatosis Sick sinus syndrome Pneumonia Cor pulmonale Atrial septal defect Digoxin Head injury

 

Send us your feedback about this question.

Single Best Answer QuestionMaximum Mark:   1Mark Awarded:   0Time taken to answer:   3 minutes 46 seconds.Main QuestionNICE 2006 guidelines recommend rate control for persistent atrial fibrillation in the following patients except:AnswersYour answer:E. Secondary to a treated precipitant (e.g. electrolyte disturbance)

Correct Answer:D. Presenting for first time from lone AFAdditional Question Info

NICE 2006 (http://guidance.nice.org.uk/CG36) guidelines recommend trying rate-control first for patients with persistent AF:

Over 65 With coronary artery disease With contraindications to antiarrhythmic drugs Unsuitable for cardioversion

Patients unsuitable for cardioversion include those with: contraindications to anticoagulation; structural heart disease (e.g. large left atrium >5.5 cm, mitral stenosis) that precludes long-term maintenance of sinus rhythm; a long duration of AF (usually >12 months); a history of multiple failed attempts at cardioversion and/or relapses, even with concomitant use of antiarrhythmic drugs or non-pharmacological approaches; an ongoing but reversible cause of AF (e.g. thyrotoxicosis). 

Send us your feedback about this question.

Single Best Answer QuestionMaximum Mark:   1

Page 16: cardiovascular

Mark Awarded:   0Time taken to answer:   1 minutes 24 seconds.Main QuestionHow best is a Pericardial friction rub heard during auscultation?AnswersYour answer:D. Lying on left side with patient breathing out.

Correct Answer:C. Left lower sternal border with the patient upright, leaning forward and breathing out.Additional Question Info

Pericardial friction rub is the most important physical sign of acute pericarditis and may have up to 3 components per cardiac cycle: high-pitched, scratching, and grating.

It can sometimes be elicited only when firm pressure with the diaphragm of the stethoscope is applied to the chest wall at the left lower sternal border. The pericardial friction rub is heard most frequently during expiration with the patient upright and leaning forward. 

Send us your feedback about this question.

Single Best Answer QuestionMaximum Mark:   1Mark Awarded:   1Time taken to answer:   0 minutes 41 seconds.Main QuestionWhat is the commonest organism responsible for subacute bacterial endocarditis?AnswersYour answer:B. Streptococcus viridians

Correct Answer:Your answer is correct!Additional Question Info

Streptococcus viridans accounts for majority of cases of subacute infective endocarditis.

Most subacute disease caused by Streptococcus viridans infection is related to dental disease. However, most cases are not caused by dental procedures but by transient bacteraemias caused by gingivitis. 

Send us your feedback about this question.

Extended Matching QuestionMaximum Mark:   3Mark Awarded:   1Time taken to answer:   3 minutes 29 seconds.Main QuestionFor each patient below, choose the most appropriate ECG change.Select ONE option only from the list.

Page 17: cardiovascular

Each option may be selected once, more than once or not at all.Answers

1. A 55 year old female with breast malignancy presents with acute chest pain, haemoptysis and shortness of breath. Her saturations are 80% on air.

Your Answer:B. T wave inversion, ST elevationCorrect Answer:C. Sinus tachycardia, RAD & RBBB

2. A 65 year old male with a history of AF complains of objects appearing green or yellow.

Your Answer:C. Sinus tachycardia, RAD & RBBBCorrect Answer:F. ST depression and inverted T waves in V5-6 (reverse tick).

3. A 70 year old female with a history of hypertension and heart failure complains of muscle weakness and cramps.

Your Answer:E. Small T waves, prominent U waves, prolonged PR interval, depressed ST segmentCorrect Answer:Your answer is correct!

Additional Question InfoQ1 - C

This patient has had a pulmonary embolism. The commonest change is sinus tachycardia. There may be RAD, RBBB, R ventricular strain (RAD, dominant R wave & T wave inversion/ST depression in V1 & V2. leads II, III & avF may show similar changes). Rarely SIQIIITIII may occur (deep S waves in I,

pathological Q waves in III and inverted T waves in III).  Q2 - F

This patient has digoxin toxicity. Visual aberration often is an early indication of digitalis toxicity. Yellow-green distortion is most common, but red, brown, blue, and white also occur. Drug intoxication also may cause snowy vision, photophobia, photopsia, and decreased visual acuity.

ECG changes include ST depression and inverted T waves in V5-6 (reversed tick). Any arrhythmia may occur (ventricular ectopics and nodal bradycardia are common).

Q3 - E

This patient has symptoms of hypokalaemia secondary to diuretic usage for her hypertension and heart failure.

ECG shows small or flattened T waves, prominent U waves (see arrows), prolonged PR interval, and occasionally depressed ST segments.

Page 18: cardiovascular

 

Send us your feedback about this question.

Single Best Answer QuestionMaximum Mark:   1Mark Awarded:   0Time taken to answer:   2 minutes 11 seconds.Main QuestionNICE 2006 guidelines recommend rhythm control for persistent atrial fibrillation in the following patients except:AnswersYour answer:D. Presenting for first time from lone AF

Correct Answer:A. >65 yearsAdditional Question Info

NICE 2006 (http://guidance.nice.org.uk/CG36) guidelines recommend trying rhythm-control first for patients with persistent AF:

Who are symptomatic Who are younger (< 65y.o) Presenting for the first time with lone AF Secondary to treated or corrected precipitant

 

Send us your feedback about this question.

Page 19: cardiovascular

Single Best Answer QuestionMaximum Mark:   1Mark Awarded:   0Time taken to answer:   1 minutes 9 seconds.Main QuestionAtrial fibrillation NICE 2006 guidelines suggests thromboprophylaxis for persistent atrial fibrillation with Aspirin 75-300mg is acceptable in the following patients except:AnswersYour answer:E. Contraindications to warfarin

Correct Answer:D. Previous TIA, stroke or thromboembolic event.Additional Question Info

NICE 2006 (http://guidance.nice.org.uk/CG36) guidelines recommend warfarin (INR 2-3) in:

Previous ischaemic stroke/TIA or thromboembolic event Age ≥75 with hypertension, diabetes or vascular disease Clinical evidence of valve disease or heart failure or impaired LV function on

Echocardiography

And Aspirin (75-300mg) may be considered in:

Age <65 with no moderate or high risk factors. Age ≥65 with no high risk factors Age <75 with hypertension, diabetes or vascular disease Contraindications to warfarin

 

Send us your feedback about this question.

Single Best Answer QuestionMaximum Mark:   1Mark Awarded:   1Time taken to answer:   1 minutes 7 seconds.Main QuestionThe following ECG changes may occur with Sarcoidosis:AnswersYour answer:E. All of the above

Correct Answer:Your answer is correct!Additional Question Info

The following ECG changes may all occur with Sarcoidosis:

1. Signs of hypercalcemia (eg, decreased QT interval) 2. Ventricular tachycardia

Page 20: cardiovascular

3. Bundle-branch block or complete heart blocks 4. ST elevation, PR depression caused by pericarditis, or ST elevation caused by ventricular aneurysm 

Send us your feedback about this question.

Single Best Answer QuestionMaximum Mark:   1Mark Awarded:   0Time taken to answer:   1 minutes 27 seconds.Main QuestionA 55 year old female is acutely short of breath. She is finding it difficult to talk, her RR is 36, O2 sats 85% on air, HR 130, BP 110/80. On auscultation a gallop rhythm and fine inspiratory bibasal crepitations can be heard. ECG shows left axis deviation and left ventricular hypertrophy. CXR shows cardiomegaly, blunted costophrenic angles and dilated prominent upper lobe vessels. Initial emergency management would comprise which of the following?

AnswersYour answer:D. Needle pericardiocentesis

Correct Answer:B. Diamorphine, IV furosemide & GTNAdditional Question Info

She has Acute heart failure.

Initial emergency management consists of: Sitting patient upright 100% Oxygen 2.5-5mg IV Diamorphine 40-80mg IV furosemide Two puffs SL GTN or 0.6mg tablets SL (only if systolic BP>90mmHg)

 

Send us your feedback about this question.

Single Best Answer QuestionMaximum Mark:   1Mark Awarded:   1Time taken to answer:   0 minutes 38 seconds.Main QuestionA 60 year old female presents to A&E short of breath, dizziness and complaining of chest pain. Her HR is 40 and her BP is 90/50mmHg. The ECG shows no relation between the atrial and slower ventricular activity. What drug would you administer?

Page 21: cardiovascular

AnswersYour answer:A. Atropine

Correct Answer:Your answer is correct!Additional Question Info

She has Complete or 3rd degree heart block.

The goal of therapy is to improve conduction through the AV node by reducing vagal tone via atropine-induced receptor blockade (Atropine 500mcg IV). 

Send us your feedback about this question.

Single Best Answer QuestionMaximum Mark:   1Mark Awarded:   0Time taken to answer:   0 minutes 43 seconds.Main QuestionA 55 year old Caucasian male sees you in your GP surgery. You measure the BP at 150/90mmHg. What do you do?AnswersYour answer:C. Ask him to measure his BP at home with a home monitoring device.

Correct Answer:B. Ask him to return in 1 month for another BP measurement.Additional Question Info

To identify Hypertension (persistent raised blood pressure above 140/90 mmHg), the recommendation is to ask the patient to return for at least two subsequent clinics where blood pressure is assessed from two readings under the best conditions available. Home monitoring is not advised.

http://guidance.nice.org.uk/CG34 

Send us your feedback about this question.

Single Best Answer QuestionMaximum Mark:   1Mark Awarded:   0

Page 22: cardiovascular

Time taken to answer:   2 minutes 18 seconds.Main QuestionYou review a 50 year old Caucasian female in your GP surgery. You prescribed Enalapril for newly diagnosed hypertension. Her BP is still elevated. According to NICE guidelines what medication should you consider next?AnswersYour answer:B. Add atenolol

Correct Answer:C. Add amlodopineAdditional Question Info

NICE guidelines advise adding a Calcium channel blocker or thiazide diuretic if the ACE-I fails to control hypertension in Caucasians under 55 years.

http://guidance.nice.org.uk/CG34. 

Send us your feedback about this question.

Single Best Answer QuestionMaximum Mark:   1Mark Awarded:   0Time taken to answer:   5 minutes 32 seconds.Main QuestionAll of the following are signs associated with cardiac tamponade except?AnswersYour answer:B. An exaggeration (>12 mm Hg) of the normal inspiratory decrease in systemic blood pressure.

Correct Answer:E. Breathing becomes progressively deeper & then shallower in cycles.Additional Question Info

Breathing becomes progressively deeper & then shallower in cycles describes Cheyne-Stokes breathing which occurs in brainstem lesions or raised intracranial pressure.

Cardiac tamponade is caused by the accumulation of fluid in the pericardial space, resulting in reduced ventricular filling and subsequent hemodynamic compromise.The pericardial space normally contains 20-50 mL of fluid. The amount of fluid needed to impair diastolic filling of the heart depends on the rate of accumulation and the compliance of the pericardium. Rapid accumulation of as little as 150 mL can severely impede cardiac output, whereas 1000 mL of fluid may accumulate over a longer period without any significant effect on diastolic filling due to adaptive stretching of the pericardium over time.

Patients present with dyspnoea, tachycardia, and tachypnoea. Cold and clammy extremities from hypoperfusion are also observed in some patients. Diminished heart sounds and a pericardial friction rub are present in approximately one third of patients.

Beck’s triad refers to increased jugular venous pressure, hypotension, and diminished heart sounds. Pulsus paradoxus is an exaggeration (>12 mm Hg or 9%) of the normal inspiratory decrease in systemic blood pressure. Kussmaul sign is a paradoxical increase in venous distention and pressure during inspiration. Ewart sign or Pins sign is observed in patients with large pericardial effusions and is described as an area of dullness, with bronchial breath sounds and bronchophony below the angle of the left scapula.

Page 23: cardiovascular

Chest radiography may show cardiomegaly, water bottle–shaped heart or pericardial calcifications. ECG may show sinus tachycardia, low-voltage QRS complexes, electrical alternans (alternation of QRS complexes, usually in a 2:1 ratio) and PR segment depression. Although echocardiography provides useful information, Cardiac tamponade is a clinical diagnosis. Treatment is Pericardiocentesis. 

Send us your feedback about this question.

Single Best Answer QuestionMaximum Mark:   1Mark Awarded:   1Time taken to answer:   1 minutes 17 seconds.Main QuestionPulsus paradoxus is NOT a feature in which of the following conditions?AnswersYour answer:A. Aortic regurgitation

Correct Answer:Your answer is correct!Additional Question Info

Pulsus paradoxus is an exaggeration ( >12 mm Hg or 9%) of the normal inspiratory decrease in systemic blood pressure.

It is seen in constrictive pericarditis, severe obstructive pulmonary disease, restrictive cardiomyopathy, pulmonary embolism, rapid and laboured breathing, and right ventricular infarction with shock.

Pulsus paradoxus is absent in patients with markedly elevated LV diastolic pressures, atrial septal defect, pulmonary hypertension, and aortic regurgitation. 

Send us your feedback about this question.

Single Best Answer QuestionMaximum Mark:   1Mark Awarded:   0Time taken to answer:   1 minutes 43 seconds.Main QuestionA 45 year old man is receiving thrombolytic treatment for a myocardial infarction on the coronary care unit. He suddenly becomes unresponsive and the cardiac monitor starts alarming to which you rush over from the doctor’s office on the unit. He has no pulse and the monitor shows he is in ventricular tachycardia. The most appropriate management would be?AnswersYour answer:C. Precordial thump followed by CPR

Correct Answer:A. DefibrillationAdditional Question Info

Page 24: cardiovascular

Pulseless VT is an indication for defribillation.

The ACLS protocol states that the precordial thump is only performed if :

(1) Witnessed arrest ; (2) Pulseless;(3) No defribillator available

See http://www.resus.org.uk/pages/gl5algos.htm 

Send us your feedback about this question.

Single Best Answer QuestionMaximum Mark:   1Mark Awarded:   0Time taken to answer:   2 minutes 16 seconds.Main QuestionThe Cardiovascular Disease Risk Prediction Charts can estimate an individual’s CVD risk over 10 years bu utilising all of the following risk factors except:AnswersYour answer:E. Total cholesterol:HDL ratio

Correct Answer:B. Diabetic statusAdditional Question Info

The Joint British Society risk charts have recently been revised. Diabetes is no longer considered a “risk factor” for calculation as it is assumed all diabetics are at high risk. Similarly the charts should not be used in those coronary heart disease or other major atherosclerotic disease, familial hypercholesterolaemia or other inherited dyslipidaemias, renal dysfunction including diabetic nephropathy.

See http://www.bhsoc.org/Cardiovascular_Risk_Prediction_Chart.stm 

Send us your feedback about this question.

Single Best Answer QuestionMaximum Mark:   1Mark Awarded:   0Time taken to answer:   3 minutes 14 seconds.Main QuestionA 40 year old male presents with sudden shortness of breath and chest pain with a history of being treated by his GP for a chest infection in the last few days. His pulse is irregularly irregular and ECG confirms fast AF at a rate of 160/min. What would be the most appropriate management?AnswersYour answer:A. Heparin & warfarin anticoagulation

Page 25: cardiovascular

Correct Answer:C. Heparin & cardioversionAdditional Question Info

Indications for rhythm control include patients who are symptomatic, who are younger, presenting for the first time with lone AF, secondary to a treated or corrected precipitant, or have congestive heart failure.

NICE guidelines (http://guidance.nice.org.uk/CG36) recommend heparinisation & (electrical or pharmacological) cardioversion if AF onset was in the last 48 hours.If AF onset was before 48 hours then administer at least 3 weeks therapeutic anticoagulation prior to cardioversion or perform TOE-guided cardioversion, depending on preference, contraindications and practicalities. 

Send us your feedback about this question.

Single Best Answer QuestionMaximum Mark:   1Mark Awarded:   0Time taken to answer:   1 minutes 58 seconds.Main QuestionThe commonest anterior mediastinal tumour is?AnswersYour answer:B. Lymphoma

Correct Answer:C. ThymomaAdditional Question Info

Thymoma is the commonest neoplasm of the mediastinum. The thymus is a lymphoid organ located behind the sternum in front of the great vessels. In early life, the thymus is responsible for the development of T-lymphpcytes and reaches its maximum weight at puberty and undergoes involution thereafter. The thymus is composed predominantly of epithelial cells and lymphocytes. Peak incidence occurs in the fourth to fifth decade of life. No sexual predilection exists.

A third of all people with a thymoma have symptoms due to compression of the surrounding organs - superior vena cava syndrome, dysphagia, cough or chest pain.

A third have a thymoma detected because they have an associated autoimmune disorder, usually myasthenia gravis (patients with myasthenia are routinely screened for thymoma). Other associated autoimmune conditions are pure red cell aplasia and Good\'s syndrome (thymoma with combined immunodeficiency and hypogammaglobulinemia).

A third are asymptomatic and their thymoma is identified on a chest X-ray or CT scan performed for an unrelated problem. CT Scan is used to estimate the size and extent of the tumor, and can be biopsied with a CT-guided needle. Thymoma originates from the epithelial cell population in the thymus. It can be staged with the Masaoka Staging System. Surgery is the mainstay of treatment ± adjunctive chemo or radiotherapy. 

Send us your feedback about this question.

Page 26: cardiovascular

Extended Matching QuestionMaximum Mark:   3Mark Awarded:   3Time taken to answer:   3 minutes 42 seconds.Main QuestionFor each patient below, choose the most appropriate murmur. Select ONE option only from the list. Each option may be selected once, more than once or not at all.Answers

1. A 60 year old female presents with chest pain and shortness of breath on exertion. On examination the carotid pulse is noted to be slow rising.

Your Answer:G. Ejection systolic murmurCorrect Answer:Your answer is correct!

2. A 60 year old male presents with exertional dyspnea, orthopnea, and paroxysmal nocturnal dyspnea. His pulse pressure is noted to be widened and his pulse is collapsing.

Your Answer:E. Early diastolic murmurCorrect Answer:Your answer is correct!

3. A 40 year old female with a history of rheumatic fever presents hoarseness, persistent cough and shortness of breath especially when walking fast. On examination she has a malar flush, is noted to be in AF, and has a tapping non-displaced apex beat.

Your Answer:D. Mid-diastolic murmurCorrect Answer:Your answer is correct!

Additional Question InfoQ1 - G

The classic crescendo-decrescendo ejection systolic murmur of Aortic Stenosis (AS) is best heard at the second intercostal space in the right upper sternal border; it is harsh at the base and radiates to both carotid arteries. However, it may be more prominent at the apex in elderly persons with calcific AS due to radiation of the high-frequency components of the murmur to the apex (Gallavardin phenomenon) leading to its misinterpretation as a murmur of MR.Q2 - E

The typical diastolic murmur of Aortic Regurgitation (AR) has a decrescendo shape. A high-frequency early diastolic murmur often occurs in mild AR, whereas a rough holodiastolic or decrescendo diastolic murmur occurs more commonly in severe AR. The volume and velocity of blood across the incompetent aortic valve tapers off in mid-to-last diastole as the aortic and LV pressures equilibrate.

The diastolic murmur of AR is usually best heard adjacent to the sternum in the second to fourth left intercostal space. A concomitant systolic ejection murmur is common in moderate-to-severe AR due to the increased volume of blood flowing across the aortic valve. The murmur associated with acute AR may not be impressive. If cardiac decompensation is present, the diastolic murmur of acute AR may be very soft and surprisingly short.Q3 - D

Page 27: cardiovascular

The diastolic murmur of mitral stenosis is of low pitch, rumbling in character, and best heard at the apex with the patient in the left lateral position. It commences after the opening snap and the duration of the murmur correlates with the severity of the stenosis. It is absent in persons with severe disease. The murmur is accentuated by exercise, cough, and amyl nitrate, whereas it decreases with rest and with Valsalva maneuver.

In patients with sinus rhythm, the murmur increases intensity in late diastole (so called, presystolic accentuation) due to increased flow across the stenotic mitral valve as a result of left atrial contraction. Hoarseness can develop from compression of the left recurrent laryngeal nerve against the pulmonary artery by an enlarged left atrium. Also, compression of bronchi by the enlarged left atrium can cause a persistent cough.  

Send us your feedback about this question.

Single Best Answer QuestionMaximum Mark:   1Mark Awarded:   1Time taken to answer:   2 minutes 48 seconds.Main QuestionA 12 year old girl complains of recurrent episodes of light-headedness and giddiness to the point where she feels she almost faints. ECG shows a shortened PR interval (<120 ms), widened QRS complex of longer than 120 milliseconds with a slurred onset of the QRS waveform producing a delta wave in the early part of QRS. What is the most likely diagnosis?

AnswersYour answer:E. Wolff-Parkinson-White syndrome

Correct Answer:Your answer is correct!Additional Question Info

Wolff-Parkinson-White (WPW) syndrome is a congenital abnormality with abnormal conductive tissue between the atria and the ventricles that is often associated with supraventricular tachycardia (SVT). Activation of the ventricles occur earlier than anticipated (preexcitation) because of conduction of an atrial impulse not by means of the normal conduction system, but via an extra atrioventricular (AV) muscular connection, termed an accessory pathway, that bypasses the AV node.

Classic ECG findings include a shortened PR interval (<120 ms), widened QRS complex of longer than 120 milliseconds with a slurred onset of the QRS waveform producing a delta wave in the early part of QRS and secondary ST-T wave changes.

Patients are potentially at increased risk of dangerous ventricular arrhythmias due to extremely fast

Page 28: cardiovascular

conduction across the bypass tract if they develop fast supraventricular arrhythmias, such as atrial flutter or fibrillation. Only a small number of patients with WPW syndrome are at risk for sudden death. In these patients, cardiac electrophysiologic studies and radiofrequency catheter ablation may be curative. Other presentations include symptomatic SVT, which can also be cured by catheter ablation. Asymptomatic patients, who form the majority, may merely need periodic observation.

In Lown-Ganong-Levine syndrome, patients have a short PR interval and SVT, but no delta wave. 

Send us your feedback about this question.

Single Best Answer QuestionMaximum Mark:   1Mark Awarded:   1Time taken to answer:   1 minutes 17 seconds.Main QuestionA 60 year old female with advanced breast cancer presents with a swollen warm left lower limb. On examination she is tachycardic and pain is elicited on dorsiflexion of the foot. Which of the following investigations would you request?AnswersYour answer:E. Duplex ultrasound

Correct Answer:Your answer is correct!Additional Question Info

This patient most likely has a DVT.

Duplex ultrasound combines doppler flow information and conventional imaging information, sometimes called B-mode, to visualise the structure of blood vessels.

Duplex ultrasound shows how blood is flowing through vessels and measures the speed of the flow of blood.  It can also be useful to estimate the diameter of a blood vessel as well as the amount of obstruction, if any, in the blood vessel. 

Send us your feedback about this question.

Single Best Answer QuestionMaximum Mark:   1Mark Awarded:   1Time taken to answer:   1 minutes 14 seconds.Main QuestionWhich investigation is definitive for a deep venous thrombosis?AnswersYour answer:B. Venogram

Correct Answer:Your answer is correct!Additional Question Info

Page 29: cardiovascular

Ascending venography with contrast dye is the definitive test for DVT but it is an invasive test and hence rarely used.

In reality D-Dimer & Duplex ultrasound are currently the investigations of choice. 

Send us your feedback about this question.

Single Best Answer QuestionMaximum Mark:   1Mark Awarded:   1Time taken to answer:   2 minutes 8 seconds.Main QuestionWhich of the following may cause a rise in D-Dimer levels?AnswersYour answer:E. All of the above

Correct Answer:Your answer is correct!Additional Question Info

D-Dimer is a degradation product of fibrin indicating intravascular coagulation & thrombosis. Normal levels are <400-500 ng/ml. Its use is indicated in suspected DVT or PE and when used in conjunction with other non-invasive studies it can reduce the need for invasive venograms.

For thrombosis sensitivity 93% & specificity 25%. This means that if the result is negative the probability of thrombosis is very small. A negative D-dimer assay result rules out DVT in patients with low-to-moderate risk and a Wells DVT score less than 2.

However a positive result does not imply the presence of thrombosis as specificity is low. Instead, it is used as an indication for further investigation with Duplex ultrasound. All patients with a positive D-dimer assay result and all patients with a moderate-to-high risk of DVT (Wells DVT score >2) require a duplex ultrasonography.

Causes of increased D-Dimer (not specific) are pulmonary embolism, deep vein thrombosis, recent surgery (within prior 1 week), myocardial infarction, infection or sepsis, cancer, concurrent systemic illness & oral anticoagulant use. 

Send us your feedback about this question.

Extended Matching QuestionMaximum Mark:   3Mark Awarded:   1Time taken to answer:   3 minutes 6 seconds.Main QuestionFor each description below, choose the most appropriate eponymous sign. Select ONE option only from the list. Each option may be selected once, more than once or not at all.Answers

Page 30: cardiovascular

1. Pulse is of the water-hammer or collapsing type, with abrupt distention and quick collapse.

Your Answer:B. Corrigan signCorrect Answer:Your answer is correct!

2. Patient’s head frequently bobs with each heartbeat.

Your Answer:G. Traube signCorrect Answer:A. de Musset sign

3. Booming systolic and diastolic “pistol-shot” sounds heard over the femoral artery.

Your Answer:D. Hill signCorrect Answer:G. Traube sign

Additional Question InfoQ1 - B

Corrigan pulse is when patients' pulses are of the water-hammer or collapsing type, with abrupt distention and quick collapse.Q2 - A

De Musset sign is when patients' heads frequently bob with each heartbeat. .Q3 - G

Traube sign (also called pistol-shot sounds) refers to booming systolic and diastolic sounds heard over the femoral artery.

De Musset sign is when patients' heads frequently bob with each heartbeat.

Corrigan pulse is when patients' pulses are of the water-hammer or collapsing type, with abrupt distention and quick collapse.

Quincke sign is when light transmitted through the patient's fingertip shows capillary pulsations.

Hill sign is when popliteal cuff systolic pressure exceeds brachial cuff pressure by more than 40 mm Hg.

Duroziez sign is when a systolic murmur is heard over the femoral artery when compressed proximally and when a diastolic murmur is heard when the femoral artery is compressed distally.

Müller sign is systolic pulsations of the uvula.

Antegrade flow across a partially closed mitral valve is thought to cause an Austin Flint murmur, which is a mid- and late-diastolic apical low-frequency murmur or rumble. The rumble occurs during premature closure of the mitral valve, which occurs when LV diastolic pressure is rising rapidly because of severe aortic reflux. Its presence indicates severe AR. 

Send us your feedback about this question.

Extended Matching QuestionMaximum Mark:   3Mark Awarded:   2Time taken to answer:   4 minutes 36 seconds.Main Question

Page 31: cardiovascular

For each ECG description below, choose the most appropriate diagnosis.Select ONE option only from the list.Each option may be selected once, more than once or not at all.Answers

1. ST elevation in leads II, III & aVF with ST depression in leads V1-6

Your Answer:F. Inferior myocardial infarctionCorrect Answer:Your answer is correct!

2. Tall R waves in V1-2 with ST depression in V1-3.

Your Answer:G. Right ventrical myocardial infarctionCorrect Answer:D. Posterior myocardial infarction

3. ST elevation in leads V1-6, I and aVL. Maximum elevation in lead V3, maximal depression in lead III.

Your Answer:E. Anterior myocardial infarctionCorrect Answer:Your answer is correct!

Additional Question InfoQ1 -F

The inferior part of the heart lies on the diaphragm and is supplied by the right coronary artery in most patients.

http://www.ecglibrary.com/infmi.html

Q2 - D

The posterior wall is usually supplied by the left circumflex coronary. Because no leads "look" at the posterior wall in a normal ECG, no leads will show ST-elevation in a posterior MI. The ST depressions in V1-V3 are in actual fact mirrored ST elevations and the high R-waves are the Q-waves of the infarct. To be able to confirm a posterior-infarct, leads V7, V8 and V9 may be helpful which are horizontally placed from V6 to the back.

http://www.ecglibrary.com/postlat.html

Q3 - E

Anterior MI can involve the anterior part of the ventricular septum and results from occlusion of the left anterior descending artery.

http://www.ecglibrary.com/ami.html

 

Send us your feedback about this question.

Extended Matching QuestionMaximum Mark:   3Mark Awarded:   3Time taken to answer:   744 minutes 33 seconds.

Page 32: cardiovascular

Main QuestionFor each patient below, choose the most appropriate diagnosis. Select ONE option only from the list. Each option may be selected once, more than once or not at all.Answers

1. A 65 year old female collapses suddenly 5 days post inferior MI and dies despite resuscitative attempts to revive her. The nurse looking after her remarks that she noticed her neck veins were quite prominent prior to collapse.

Your Answer:F. Ventricular free wall ruptureCorrect Answer:Your answer is correct!

2. A 70 year old male is 4 hours post ST elevation MI and has received thrombolytic therapy. Whilst on the coronary care unit he suddenly became unresponsive and required defibrillation to correct his arrhythmia.

Your Answer:D. Ventricular fibrillationCorrect Answer:Your answer is correct!

3. A 60 year old female is 1 week post ST elevation MI. She complains of shortness of breath & on examination there is a pansystolic murmur best heard at the apex and a palpable thrill.

Your Answer:H. Mitral regurgitationCorrect Answer:Your answer is correct!

Additional Question InfoQ1 - F

Ventricular free wall rupture (VFWR) is the most serious complication after MI. VFWR is usually associated with large transmural infarctions and antecedent infarct expansion. It is the most common cause of death, second only to LV failure, and it accounts for 15-30% of the deaths associated with AMI. Incontrovertibly the most catastrophic of mechanical complications, VFWR leads to acute haemopericardium and death from cardiac tamponade.

Risk factors of VFWR include advanced age (>60 y), hypertension, female sex, and no previous MIs. Patients with a history of angina pectoris, previous AMI, multivessel coronary disease, and CHF are less likely than others to develop VFWR of the LV because they develop collaterals and ischemic preconditioning.Q2 - D

Cardiac arrhythmias are not uncommon during and immediately after an MI. Of all patients who have an MI, about 90% develop some form of cardiac arrhythmia. The incidence increases with an ST-elevation myocardial infarction (STEMI). The incidence of primary ventricular fibrillation (VF) (5%) is greatest in the first hour after the onset of infarct; thereafter, the incidence rapidly declines. Secondary or late VF occurring more than 48 hours after an MI is usually associated with pump failure and cardiogenic shock.

Treatment for VF is unsynchronized electrical countershock with at least 200-300 J (or biphasic energy equivalent) administered as rapidly as possible. Each minute after the onset of uncorrected VF is associated a 10% decrease in the likelihood of survival. Restoration of synchronous cardiac electrical activity without the return of effective contraction (ie, electromechanical dissociation, or pulseless electrical activity) is generally due to extensive myocardial ischemia and/or necrosis or cardiac rupture.Q3 - H

Mitral Regurgitation (MR) is a common complication of MI that results from local and global left ventricle remodelling and that is an independent predictor of heart failure and death. MR typically occurs 7-10 days after an MI, though this onset may vary according to the mechanism of MR. Papillary muscle rupture resulting in MR usually occurs within a fortnight. Mild-to-moderate MR is often clinically silent and detected on Doppler echocardiography performed during the early phase of MI.

In this case, MR rarely causes haemodynamic compromise. Severe acute MR that results from the

Page 33: cardiovascular

rupture of papillary muscles or chordae tendineae results in abrupt haemodynamic deterioration with cardiogenic shock. Rapid diagnosis, haemodynamic stabilization, and prompt surgical intervention are needed because acute severe MR is associated with a high mortality rate. 

Send us your feedback about this question.

Extended Matching QuestionMaximum Mark:   3Mark Awarded:   2Time taken to answer:   1 minutes 54 seconds.Main QuestionFor each ECG description below, choose the most appropriate diagnosis. Select ONE option only from the list. Each option may be selected once, more than once or not at all.Answers

1. Progressive increase in PR interval until there is a non-conducted P wave.

Your Answer:B. Mobitz type I AV blockCorrect Answer:Your answer is correct!

2. No relation between the atrial and slower ventricular activity.

Your Answer:E. Atrial fibrillationCorrect Answer:D. Complete AV block

3. Prolonged PR interval > 0.2 sec.

Your Answer:A. First degree AV blockCorrect Answer:Your answer is correct!

Additional Question InfoQ1 - B

Second-degree heart (atrioventricular) block, occurs when some atrial impulses are not conducted to the ventricles. Electrocardiographically, some P waves are not followed by a QRS complex. Second-degree AV block is composed of 2 types: Mobitz I or Wenckebach block, and Mobitz II. The Mobitz I second-degree AV block is characterized by a progressive prolongation of the PR interval, which results in a progressive shortening of the R-R interval. Ultimately, the atrial impulse fails to conduct, a QRS complex is not generated, and there is no ventricular contraction.

Q2 - D

Complete heart block, also referred to as third-degree heart (AV) block, is a disorder of the cardiac conduction system where there is no conduction through the AV node. Therefore, complete dissociation of the atrial and ventricular activity exists.

Page 34: cardiovascular

Q3 - A

On an ECG the PR interval should be between 0.12 and 0.2 sec. First-degree heart (AV) block is defined as prolongation of the PR interval greater than 0.2 sec. First-degree heart block is considered "marked" when the PR exceeds 0.3 sec.

 

Send us your feedback about this question.

Extended Matching QuestionMaximum Mark:   3Mark Awarded:   2Time taken to answer:   2 minutes 10 seconds.Main QuestionFor each type of chest pain below, choose the most appropriate diagnosis. Select ONE option only from the list.Each option may be selected once, more than once or not at all.Answers

1. Relieved by sitting up and leaning forward and is intensified by lying supine.

Your Answer:A. Pericardial effusionCorrect Answer:Your answer is correct!

2. Tearing pain in the intrascapular area

Your Answer:D. Aortic arch dissectionCorrect Answer:E. Descending aorta dissection

3. Exacerbated by trunk movement, deep inspiration, and/or exertion and relieved with decreased movement, quiet breathing.

Your Answer:H. CostochondritisCorrect Answer:Your answer is correct!

Additional Question InfoQ1 - A

Characteristically, pericardial pain may be relieved by sitting up and leaning forward and is intensified by lying supine.Q2 - E

Page 35: cardiovascular

Aortic dissection is painless in about 10% of patients. Painless dissection is more common in those with neurologic complications from the dissection and those with Marfan syndrome.

The description of the pain may indicate where the dissection arises: Anterior chest pain and chest pain that mimics anterior myocardial infarction usually are

associated with anterior arch or aortic root dissection. This is caused by the dissection interrupting flow to the coronary arteries, resulting in myocardial ischemia.

Pain that is described in the neck or jaw indicates that the dissection involves the aortic arch and extends into the great vessels of the arch.

Tearing or ripping pain that is felt in the intrascapular area may indicate that the dissection involves the descending aorta. The pain typically changes as the dissection evolves.

Q3 -H

Pain in costochondritis is a sharp, nagging, aching usually well-localised pain exacerbated by trunk movement, deep inspiration, and/or exertion and lessens with decreased movement, quiet breathing, or change of position. 

Send us your feedback about this question.

Extended Matching QuestionMaximum Mark:   3Mark Awarded:   1Time taken to answer:   2 minutes 35 seconds.Main QuestionFor each patient below, choose the most appropriate option. Select ONE option only from the list. Each option may be selected once, more than once or not at all.Answers1. A patient suddenly collapses and becomes unresponsive. There is no sign of breathing and a pulse is not palpable. You immediately call for help and administer basic life support. What ratio of chest compressions : breaths do you deliver?

Your Answer:B. 15:1Correct Answer:A. 30:2

2. The resusucitation team arrive and a defibrillator/monitor is attached which shows asystole. What do you do now?

Your Answer:H. Resume CPR 30:2 for 2 minutesCorrect Answer:Your answer is correct!

3. The rhythm now changes to ventricular fibrillation. What do you do now?

Your Answer:F. 2 Shocks (150-360 J biphasic or 360 J monophasic)Correct Answer:G. 1 Shock (150-360 J biphasic or 360 J monophasic)

Additional Question InfoQ1 - A

30:2  is the current compressions : breaths ratio.

See the 2005 UK resuscitation council guidelines. Q2 - H

Asystole is not a shockable rhythm.

Page 36: cardiovascular

Q3 - G

Ventricular fibrillation is a shockable rhythm. 

Send us your feedback about this question.

Extended Matching QuestionMaximum Mark:   3Mark Awarded:   2Time taken to answer:   5 minutes 41 seconds.Main QuestionFor each patient below, choose the most appropriate management. Select ONE option only from the list. Each option may be selected once, more than once or not at all.Answers

1. A 40 year old male patient presents with shortness of breath. On examination he is tachycardic, hypotensive, tachypnoeic and his neck veins are distended. On auscultation he has diminished heart sounds and a pericardial friction rub is present.

Your Answer:D. Needle pericardiocentesisCorrect Answer:Your answer is correct!

2. A 50 year old male presents with severe central chest pain. ECG shows ST segment elevation in leads II, III & aVF. Troponin T is elevated.

Your Answer:G. ThrombolysisCorrect Answer:Your answer is correct!

3. A 40 year old afro-carribean male presents with a headache, dizziness and visual disturbances. On examination his blood pressure is 210/130mmHg and papilloedema is present on fundoscopy.

Your Answer:E. LabetalolCorrect Answer:H. Atenolol

Additional Question Info

Q1 - D

Emergency subxiphoid percutaneous drainage is a life-saving procedure for cardiac tamponade.

The subxiphoid approach is extrapleural, hence, it is the safest for blind pericardiocentesis. A 16- or 18-gauge needle is inserted at an angle of 30-45° to the skin, near the left xiphocostal angle, aiming towards the left shoulder.

Echocardiographically guided pericardiocentesis (often performed in the cardiac catheterization laboratory) is usually performed from the left intercostal space.Q2 - G

Diagnosis of MI requires Two out of the following Three:

(1) Cardiac type chest pain; (2) Evolutionary changes on serial ECGs;(3) Rise in serum cardiac markers.

The initial goals in the management of a myocardial infarction are:

(1) Prompt and accurate diagnosis;(2) Admission to a coronary care unit;

Page 37: cardiovascular

(3) Pain relief;(4) Myocardial reperfusion (thrombolysis versus primary percutaneous intervention);(5) Secondary prophylaxis.Q3 - H

Malignant hypertension is a distinctive form of hypertension characterised by vascular fibrinoid necrosis and loss of precapillary arteriolar autoregulation. It is a medical emergency as blood pressure may rise acutely - to a diastolic level greater than 130 mm Hg. The end-organ effects of hypertension may develop rapidly.

Symptoms may include visual disturbance, headaches, breathlessness. hypertensive encephalopathy (blunting of conscious level, coma, epileptic seizures).

On examination, blood pressure is elevated with a diastolic greater than 130 mm Hg, papilloedema and there may be evidence of haemorrhages and exudates visible on the retina, proteinuria, signs of cardiac failure and rarely, haemoglobinuria, jaundice and anaemia (microangiopathic haemolytic anaemia).

Treatment involves bed rest, blood pressure monitoring and aim to reduce the blood pressure to 110 mm Hg diastolic over 24 hours. Drug treatment is generally with oral therapy unless there is encephalopathy or CCF. Atenolol or long acting calcium channel blockers (avoid sublingual nifedipine as this will provoke a large drop in BP → stroke risk). If encephalopathic, use IV furosemide followed by IV labetalol or IV sodium nitroprusside. 

Send us your feedback about this question.

Extended Matching QuestionMaximum Mark:   3Mark Awarded:   2Time taken to answer:   4 minutes 56 seconds.Main QuestionFor each scenario below, choose the most appropriate investigation. Select ONE option only from the list. Each option may be selected once, more than once or not at all.Answers

1. If you suspect a patient has chronic heart failure which initial investigation(s) are advised by NICE guidelines 2003 to exclude or confirm your suspicions?

Your Answer:B. I2 lead ECG & Chest x-rayCorrect Answer:F. B-type natriuretic peptide & 12 lead ECG

2. Your initial investigation(s) from question 1 confirm your suspicions. What investigation(s) should you request next?

Your Answer:G. Transthoracic echocardiographyCorrect Answer:Your answer is correct!

3. The results of the investigation(s) from question 2 are inconclusive. What investigation(s) should your perform next?

Your Answer:H. Transoesophageal echocardiographyCorrect Answer:Your answer is correct!

Additional Question InfoQ1 - F

NICE guidelines 2003 recommends patients with suspected heart failure are investigated with ECG &

Page 38: cardiovascular

B-type natriuretic peptide (BNP) where available.

B-type natriuretic peptide(BNP), previously known as Brain natriuretic peptide is a polypeptide secreted by the ventricles of the heart in response to excessive stretching of cardiomyocytes. BNP is a very sensitive test (97%) and if negative the patient can be reassured that they do not have heart failure.

However, it is not very specific (80%) so patients who test positive need an echocardiogram to confirm the left ventricle dysfunction. Patients with left ventricular dysfunction usually have ECG abnormalities such as left ventricular hypertrophy.Q2 - G

Next investigation would be Transthoracic Doppler 2D-echocardiography.Q3 - H

If poor images are produced from transthoracic Doppler 2D-echocardiography alternative methods should be considered such as Transoesophageal echocardiography, Radionuclide imaging or Cardiac magnetic resonance imaging. 

Send us your feedback about this question.

Extended Matching QuestionMaximum Mark:   3Mark Awarded:   1Time taken to answer:   13 minutes 2 seconds.Main QuestionFor each scenario below, choose the most appropriate option. Select ONE option only from the list. Each option may be selected once, more than once or not at all.Answers

1. A 30 year old female presents with a swollen warm right leg. Her D-Dimer is negative. What would you do next?

Your Answer:B. Duplex ultrasoundCorrect Answer:E. Nothing further needs doing

2. A 50 year old male bed-bound with back pain presents with a swollen right leg that is >3cm compared with the non-swollen leg. On examination he has pitting oedema. D-Dimer is negative. What do you do next?

Your Answer:B. Duplex ultrasoundCorrect Answer:Your answer is correct!

3. A 65 year old female patient who is one week post hip replacement surgery has been diagnosed with a deep vein thrombosis. What do you do next?

Your Answer:B. Duplex ultrasoundCorrect Answer:F. Start low molecular weight heparin and warfarinise for 3mths.

Additional Question Info

Q1 - E

She has a Wells score  of <2 and has low-moderate risk of a DVT and in conjunction with a negative D-Dimer, the probability of a DVT is low.

Ref: Anand SS, Wells PS, Hunt D, Brill-Edwards P, Cook D, Ginsberg JS. Does this patient have deep vein thrombosis? JAMA 1998 Apr 8;279(14):1094-9

Page 39: cardiovascular

 Q2 - B

This gentleman has a Wells score of 3 and has a high probability of having a DVT. So despite the negative D-Dimer, he will need a Duplex ultrasound.Q3 - F

Warfarinise for:3months if post operative, 6months if no cause is found,life-long if recurrent DVT or thrombophilia. 

Send us your feedback about this question.

Single Best Answer QuestionMaximum Mark:   1Mark Awarded:   0Time taken to answer:   11 minutes 15 seconds.Main QuestionYou are working on the Medical Assessment Unit and you see a GP referral. A 55 year old female who is at high risk of DVT (Wells score >2: active lung cancer, reduced mobility due to SOB, calf diameter discrepancy > 3cm) presents with a swollen right calf. Clinically you feel she has a DVT. Her D-Dimer is 400ng/ml (cut off <220ng/ml). So you arrange an ultrasound scan which however comes back as showing no evidence of DVT. What do you do?AnswersYour answer:D. Go with your clinical acumen and commence oral anticoagulation for 3mths

Correct Answer:B. Arrange a repeat ultrasound scan in one weekAdditional Question Info

You may be familiar with the following algorithm used in many NHS hospitals. With regards to this patient a repeat ultrasound scan is the best answer.

Page 40: cardiovascular
Page 41: cardiovascular

 

Send us your feedback about this question.

Single Best Answer QuestionMaximum Mark:   1Mark Awarded:   1Time taken to answer:   1 minutes 17 seconds.Main QuestionYou are working on the Medical Assessment Unit and a health care assistant has just done an ECG on a patient and flashes the following ECG under your nose for you to have a look at. What electrolyte abnormality does the patient have?

Page 42: cardiovascular

AnswersYour answer:D. Hyperkalaemia

Correct Answer:Your answer is correct!Additional Question Info

This patient had a potassium of 9.0mmol/l.

Hyperkalaemic changes on the ECG include Tall tented T-waves, widened QRS and small P waves.

Hypokalaemic changes include small or inverted T-waves, prominent U wave (after T wave), prolonged PR interval & depressed ST segment.

Hypercalcaemic changes include a reduced QT interval.

Hypocalcaemic changes include a prolonged QT interval.   

Send us your feedback about this question.

Single Best Answer QuestionMaximum Mark:   1Mark Awarded:   1Time taken to answer:   1 minutes 41 seconds.Main QuestionA 29 year old male scaffolder presents with left sided chest pain. He admits to doing a lot of lifting the day before. His chest wall is tender to palpation. His ECG shows normal sinus

Page 43: cardiovascular

rhythm with a rate of 70. His troponin is <0.01ng/ml and his D-Dimer is 34 μg/l. His CXR is below. What is the most likely diagnosis?

AnswersYour answer:C. Costochondritis

Correct Answer:Your answer is correct!Additional Question Info

The history, examination and negative investigation results suggest he has costochondritis which is an inflammatory process of the costochondral or costosternal joints that causes localized pain and tenderness. Any of the 7 costochondral junctions may be affected, and more than 1 site is affected in 90% of cases. The second to fifth costochondral junctions most commonly are involved. The pain often comes after a period of strenuous activity and is exacerbated by moving the trunk and deep breathing. While similar, Tietze’s syndrome is not identical to costochondritis. Tietze’s syndrome is characterized by swelling of the costal cartilages, while in costochondritis there is no swelling. 

Send us your feedback about this question.

Single Best Answer QuestionMaximum Mark:   1Mark Awarded:   1Time taken to answer:   1 minutes 37 seconds.Main QuestionAn 80year old male complains of left sided chest tightness that lasted a couple of hours. He called the paramedics who gave him 300mg Aspirin & GTN at the scene. He was then transferred to A&E. He has no more chest pain now and his observations are normal. His ECG is in sinus rhythm with no acute ischaemic changes. What blood test should you request?AnswersYour answer:C. Troponin-I

Correct Answer:Your answer is correct!

Page 44: cardiovascular

Additional Question Info

Cardiac biomarkers should be measured in all patients who present with chest discomfort consistent with acute coronary syndrome.

Cardiac Troponins T and I are the preferred markers for myocardial injury because they are more sensitive and more specific than the creatine kinase MB isoenzyme (CK-MB), aspartate aminotransferase (AST) or lactate dehydrogenase (LAD), which have much lower specificity to cardiac muscle.

Patients with negative cardiac biomarkers within 6 hours of the onset of symptoms that are consistent with ACS should have biomarkers re-measured in the time frame of 8 to 12 hours after the onset of symptoms. Elevations of cardiac enzyme levels should be interpreted in the context of clinical and ECG findings.

Peak circulating enzyme levels tend to occur earlier and are often higher following successful thrombolytic therapy:

 

Send us your feedback about this question.

Single Best Answer QuestionMaximum Mark:   1Mark Awarded:   0Time taken to answer:   2 minutes 32 seconds.Main QuestionA 65 year old male presents to A&E with a history of angina presents with a history of 1 hour of crushing retrosternal chest pain radiating to his left arm. His ECG shows evidence of ST

Page 45: cardiovascular

depression in the inferior leads. What would be the most appropriate management?AnswersYour answer:B. Emergent Thrombolysis

Correct Answer:C. 12 hr Troponin-I levelsAdditional Question Info

Acute Coronary Syndrome (ACS) without ECG ST segment elevation comprises of unstable angina and NSTEMI and excludes by definition persistent ECG ST segment elevation Myocardial Infarction (STEMI).

Patients usually present with one of the following patterns of symptoms:

1. Abrupt or worsening of previous angina, with symptoms becoming more frequent, more severe or more prolonged and less responsive to nitroglycerine

2. Prolonged (> 15 minutes) angina occurring at rest.

The ECG may show:

1. ST segment depression

2. Transient ST segment elevation which resolves spontaneously or after nitroglycerine

3. T wave inversion

4. Evidence of previous myocardial infarction

5. Left bundle branch block

6. Minor non-specific changes

7. Or can be normal

But should NOT show persistent acute ST segment elevation

Initial Assessment

Clinical history and examination

ECG within 5 minutes

Base line observations blood pressure, pulse, respiration rate, bedside blood glucose & temperature

Troponin I, FBC, U&E, Glucose & lipids

Chest X-ray

Risk Assessment (GRACE score)

Initial Treatment

1. Aspirin; all patients with a confirmed acute coronary syndrome should be given Aspirin unless contraindicated; give a loading dose of 300 mgs followed by 75mgs daily

2. Clopidogrel; high risk patients and patients who are truly allergic to Aspirin give Clopidogrel, 300 mgs loading dose followed by 75 mgs daily for 12 months

3. Low molecular weight heparin (LMWH); give enoxoparin 1 mg/kg 12 hourly. In general heparin is of

Page 46: cardiovascular

benefit for at least two days and up to eight days if there is continuing ischaemia or high clinical risk

4. Nitrates oral/intravenous; there is no survival benefit from nitrate. Use it to control hypertension or to control angina pain.

Pain relief should be achieved; if oral/IV nitrates are not successful in reducing pain the use of Diamorphine is indicated. Diamorphine 2.5 to 5 mg can be given intravenously at 1mg/min.

 

Send us your feedback about this question.

Single Best Answer QuestionMaximum Mark:   1Mark Awarded:   1Time taken to answer:   1 minutes 42 seconds.Main QuestionYou are called to see a 65 year old gentleman on the ward who the nurses are concerned about as his respiratory rate is 30 and his oxygen saturations are 89% on air. He sleeps on 4 pillows at night. On auscultation there is evidence of fine bibasal crepitations. You arrange for a CXR which is shown below. What is the most likely diagnosis?

AnswersYour answer:C. Left ventricular failure

Correct Answer:Your answer is correct!Additional Question Info

Left ventricular failure presents with dyspnoea, fatigue, orthopnoea, paroxysmal nocturnal dyspnoea, nocturnal cough sometimes productive of pink frothy sputum. Classical signs on CXR are alveolar and interstitial oedema, upper lobe diversion, cardiomegaly and bilateral small pleural effusions. In the CXR shown in the question there is an obvious Kerley B line (indicative of interstitial oedema).

 

Send us your feedback about this question.

Page 47: cardiovascular

Single Best Answer QuestionMaximum Mark:   1Mark Awarded:   0Time taken to answer:   3 minutes 3 seconds.Main QuestionA 68 year old male with a background of type 2 diabetes and hypertension is referred by his GP to the Medical Assessment Unit with left sided weakness since this morning (>1hr ago). His blood pressure is 160/100mmHg. What is his ABCD2 score?AnswersYour answer:C. 2

Correct Answer:E. 7Additional Question Info

The ABCD2 score is a clinical score to determine the risk for stroke within the first two days following a transient ischemic attack (TIA). The score is based on five parameters: age, blood pressure, clinical features, duration of TIA, and presence of diabetes. Each item is scored and the results are added to a result ranging between zero and seven.

 The risk for stroke can be estimated from the ABCD2 score as follows:

 Score 1-3 (low)

2 day risk = 1.0%

Page 48: cardiovascular

7 day risk = 1.2%

 Score 4-5 (moderate)

2 day risk = 4.1%

7 day risk = 5.9%

 Score 6–7 (high)

2 day risk = 8.1%

7 day risk = 11.7%

 In this question, this man scores 7 and is at high risk for a stroke and would need to be commenced on 300mg Aspirin (if not contraindicated for a variable period between 7-10 days depending on your local hospital policy) and further investigation ASAP with CT Head and doppler studies of his carotids.

 

Send us your feedback about this question.

Single Best Answer QuestionMaximum Mark:   1Mark Awarded:   0Time taken to answer:   0 minutes 58 seconds.Main QuestionA 49 year old homeless male is brought into A&E resusc barely responsive with a temperature of 29°C. He is bradycardic with a rate of 50 and his blood pressure is not recordable. An ECG after many attempts is recorded below. What pathognomonic feature of hypothermia does it show?

Page 49: cardiovascular

AnswersYour answer:B. Delta waves

Correct Answer:C. J-wavesAdditional Question Info

The following are characteristic of the ECG changes in the hypothermic patient:  

Atrial fibrillation

PR elongation

QRS widening

QT elongation

J-waves (pathognomonic of hypothermia)

Page 50: cardiovascular

These abnormalities may proceed to arrhythmias.

 

Send us your feedback about this question.

Single Best Answer QuestionMaximum Mark:   1Mark Awarded:   1Time taken to answer:   0 minutes 50 seconds.Main QuestionAn 82 year old asian male presents with crushing left sided chest pain radiating to his neck & jaw for 2 hours. He has a strong cardiac history with a previous MI 20 years ago and CABG at around the same time. 12hr Troponin is elevated. An ECG & CXR is performed. What is the likely diagnosis?

Page 51: cardiovascular

AnswersYour answer:B. NSTEMI

Correct Answer:Your answer is correct!Additional Question Info

ECG shows normal sinus rhythm with a rate of 65bpm with RBBB and T-wave inversion in the V1 & V2 leads. There is no evidence of ST elevation or features of pericarditis or pulmonary embolism. His CXR appears normal. His 12hr troponin is elevated suggesting NSTEMI.   

Page 52: cardiovascular

Send us your feedback about this question.

Single Best Answer QuestionMaximum Mark:   1Mark Awarded:   1Time taken to answer:   1 minutes 28 seconds.Main QuestionWhen consenting a MI patient for thrombolysis – which of the following is not true?AnswersYour answer:C. Total stroke risk about 25%

Correct Answer:Your answer is correct!Additional Question Info

Risks for Tenecteplase (Metalyse):

All bleeding complications: 26%Major bleed: 5% Intracranial haemorrhage: 1%Intracranial haemorrhage (age > 75 years): ~ 2%

I have seen a number of bleeding complications from the use of thrombolysis. The “disasters” invariably occur where the consent taken was questionable (i.e., from an elderly and possibly slightly demented spouse) or from people whose English is not their first language and an interpretor was not used.

 

Send us your feedback about this question.

Single Best Answer QuestionMaximum Mark:   1Mark Awarded:   0Time taken to answer:   0 minutes 41 seconds.Main QuestionAn 82 year old female who suffers from dementia & Type 2 diabetes mellitus is brought to A&E increasingly confused and unwell. She is moaning in some distress. What does the ECG show?

Page 53: cardiovascular

AnswersYour answer:C. Lateral MI

Correct Answer:D. Anteroseptal MIAdditional Question Info

This patient had an acute anteroseptal STEMI. Due to her premorbid state it was decided that she was not a candidate for PPCI or thrombolysis. 

Send us your feedback about this question.

Extended Matching QuestionMaximum Mark:   3Mark Awarded:   1Time taken to answer:   3 minutes 34 seconds.Main QuestionFor each statement below, choose the most appropriate scoring system. Select ONE option only from the list above. Each option may be selected once, more than once or not at all.Answers

1. Used to assess risk of death in Acute Coronary Syndrome

Your Answer:C. CHADS2Correct Answer:G. GRACE

2. Used to estimate the risk of Stroke and determine degree of anticoagulation Your Answer:

Page 54: cardiovascular

neededG. GRACECorrect Answer:C. CHADS2

3. Used to determine the risk of DVT

Your Answer:F. WELLSCorrect Answer:Your answer is correct!

Additional Question Info

Q1   -   G

GRACE score factors in the Age, HR, SYS BP, Creatinine & Killip Class Heart Failure to work out the risk of dying in Acute Coronary Syndrome. See http://www.outcomes-umassmed.org/GRACE/index.cfm   

Q2   -   C

CHADS2 score estimates the risk of stroke in patients aged 65 to 95 with nonrheumatic atrial fibrillation and is used to determine the degree of anticoagulation.

Congestive heart failure (score 1)Hypertension (1)Age>75 (1)D/M (1)S2 Prior stroke or TIA (2). 

A score of 0 is low risk and aspirn is recommended. 

A score of 1 is moderate risk and either aspirin or warfarinise to INR of 2-3 is recommended depending on patient preferences.

  A score ≥2 is high risk and patients should be warfarinised to an INR of 2-3 unless

contraindicated (e.g. history of falls, clinically significant GI bleeding, inability to obtain regular INR screening, etc.)

 

Q3   -   F

In 2006, Scarvelis and Wells overviewed a set of clinical prediction rules for DVT:

Wells score or criteria: (Possible score -2 to 8)

1. Active cancer (treatment within last 6 months or palliative) -- 1 point

2. Calf swelling >3 cm compared to other calf (measured 10 cm below tibial tuberosity) -- 1 point

3. Collateral superficial veins (non-varicose) -- 1 point

4. Pitting oedema (confined to symptomatic leg) -- 1 point

5. Swelling of entire leg - 1 point

6. Localized pain along distribution of deep venous system—1 point

7. Paralysis, paresis, or recent cast immobilization of lower extremities—1 point

Page 55: cardiovascular

8. Recently bedridden > 3 days, or major surgery requiring regional or general anaesthetic in past 4 weeks—1 point

9. Alternative diagnosis at least as likely—Subtract 2 points  Interpretation:

Score of 2 or higher - deep vein thrombosis is likely. Consider imaging the leg veins.Score of less than 2 - deep vein thrombosis is unlikely. Consider blood test such as d-dimer test to further rule out deep vein thrombosis. 

 

Send us your feedback about this question.

Extended Matching QuestionMaximum Mark:   3Mark Awarded:   2Time taken to answer:   4 minutes 7 seconds.Main QuestionFor each patient below, choose the most appropriate diagnosis. Select ONE option only from the list above. Each option may be selected once, more than once or not at all.Answers

1. A 78 year old male is taken to A&E after collapsing in a restaurant. He reports that the restaurant was excessively warm, that he began to feel light-headed and nauseous. His wife says that he looked pale before collapsing. On examination there was no neurological deficit.

Your Answer:B. Vasovagal syncopeCorrect Answer:Your answer is correct!

2. A 63 year old male calls NHS Direct late at night as he became severely SOB when he lay down to sleep. He also reports coughing up pink frothy sputum but no chest pain. Paramedics were called who noted his saturations on air were 84%. They administered IV Furosemide which gave him considerable improvement. He has no medical history of note but he did report a short episode of SOB the day before whilst gardening.

Your Answer:A. Pulmonary embolismCorrect Answer:F. Flash pulmonary oedema

3. A 25 year old male warehouse worker complains of pain on the left side of his chest worse with movement. He points to his nipple area which is tender to palpation but no swelling is palpable. ECG is normal.

Your Answer:D. CostochondritisCorrect Answer:Your answer is correct!

Additional Question Info

Q1   -   B

This patient has had a vasovagal syncope. This is neurally mediated and is due to excessive parasympathetic tone and sympathetic suppression resulting in peripheral pooling of blood, bradycardia & hypotension. The factors which trigger the attack include emotion, fear and pain, heavy meals and alcohol, micturition, defecation and coughing, prolonged upright posture, a warm environment & dehydration.

The symptoms of neurocardiogenic syndrome often include a prodromal stage with weakness, nausea, sweating, pallor, visual disturbance, abdominal discomfort, headache, pins-and-needles, light-

Page 56: cardiovascular

headedness or vertigo. A witness may report that the patient appeared pale, was cold and clammy had a short seizure or became orientated rapidly upon regaining consciousness.    

Q2   -   F

Flash pulmonary oedema is rapid onset pulmonary oedema. It is most often precipitated by acute myocardial infarction or mitral regurgitation, but can be caused by aortic regurgitation, heart failure, or almost any cause of elevated left ventricular filling pressures. This patient was taken to A&E by the paramedics where his troponin was elevated. His ECG also showed new onset LBBB but no ST changes. It was assumed that he had had a myocardial event the day before whilst gardening.

Treatment of flash pulmonary oedema should be directed at the underlying cause, but the mainstays are ensuring adequate oxygenation, diuresis, and decrease of pulmonary circulation pressures. 

Q3   -   D

Costochondritis is an inflammatory process of the costochondral or costosternal joints that causes localized pain and tenderness. Any of the 7 costochondral junctions may be affected, and more than 1 site is affected in 90% of cases. The second to fifth costochondral junctions most commonly are involved. The pain often comes after a period of strenuous activity and is exacerbated by moving the trunk and deep breathing.   

 

Send us your feedback about this question.

Extended Matching QuestionMaximum Mark:   3Mark Awarded:   3Time taken to answer:   9 minutes 0 seconds.Main QuestionFor each statement below, choose the most appropriate lipid lowering drug. Select ONE option only from the list above. Each option may be selected once, more than once or not at all.Answers

1. Can predispose to gallstones by increasing biliary cholesterol excretion

Your Answer:C. CholestyramineCorrect Answer:Your answer is correct!

2. Should be considered for all patients, including the elderly, with symptomatic cardiovascular disease

Your Answer:E. SimvastatinCorrect Answer:Your answer is correct!

3. May cause severe flushing Your Answer:A. Nicotinic acidCorrect Answer:Your answer is

Page 57: cardiovascular

correct!

Additional Question Info

Q1   -   C

Cholestyramine is a bile acid sequestrant, which binds bile in the gastrointestinal tract to prevent its reabsorption which are then excreted in the faeces. When bile acids are excreted, plasma cholesterol is converted to bile acid to normalize bile acid levels. This conversion of cholesterol into bile acids lowers plasma cholesterol concentrations. Bile acid sequestrants such as cholestyramine are primarily used to treat hypercholesterolemia, but can also be used to treat the pruritus, or itching, that often occurs during liver failure due to the liver's inability to eliminate bile.

The following side effects have been noted: Most frequent: Constipation Seldom: tooth discoloration, tooth enamel erosion, and premature tooth decay, all from

prolonged oral exposure to the suspension There is also an increased risk for gallstones due to increased cholesterol concentration of bile.  

Q2   -   E

The statins (atorvastatin, fluvastatin, pravastatin, rosuvastatin, and simvastatin) competitively inhibit HMG CoA (3-hydroxy-3-methylglutaryl coenzyme A) reductase, an enzyme involved in cholesterol synthesis, especially in the liver. Statins are more effective than other lipid-regulating drugs at lowering LDL-cholesterol concentration but they are less effective than the fibrates in reducing triglyceride concentration.

Statins reduce cardiovascular disease events and total mortality irrespective of the initial cholesterol concentration. Statins should be considered for all patients, including the elderly, with symptomatic cardiovascular disease such as those with coronary heart disease (including history of angina or acute myocardial infarction), occlusive arterial disease (including peripheral vascular disease, non-haemorrhagic stroke, or transient ischaemic attacks). In patients with diabetes mellitus, the risk of developing cardiovascular disease depends on the duration and complications of diabetes, age, and concomitant risk factors.

Statin therapy should be considered for all patients over 40 years with diabetes mellitus (type 1 and 2). In younger patients with diabetes, treatment with a statin should be considered if there is target-organ damage, poor glycaemic control (HbA1c greater than 9%), low HDL-cholesterol and raised triglyceride concentration, hypertension, or a family history of premature cardiovascular disease.

Statins are also used for the prevention of cardiovascular disease events in asymptomatic individuals who are at increased risk (see Lipid-regulating drugs). Statin treatment should also be considered if the total cholesterol concentration to HDL-cholesterol ratio exceeds 6. 

Q3   -   A

The value of Nicotinic acid is limited by its side-effects, especially vasodilatation. It lowers both cholesterol and triglyceride concentrations by inhibiting synthesis & it also increases HDL-cholesterol. Nicotinic acid is licensed for use with a statin if the statin alone cannot adequately control dyslipidaemia (raised LDL-cholesterol, triglyceridaemia, and low HDL-cholesterol). It can be used alone if the patient is intolerant of statins. A preparation combining laropiprant with nicotinic acid (Tredaptive®) is available (laropiprant has no lipid-regulating effect, but reduces the symptoms of flushing associated with nicotinic acid).  

Page 58: cardiovascular

 

Send us your feedback about this question.

Extended Matching QuestionMaximum Mark:   3Mark Awarded:   3Time taken to answer:   3 minutes 6 seconds.Main QuestionFor each scenario below, choose the most appropriate action. Select ONE option only from the list above. Each option may be selected once, more than once or not at all.Answers

1. A 65 year old male with a strong cardiac history is out celebrating his retirement. He develops sudden onset crushing chest pain about 15minutes prior to being rushed into A&E. He is visibly pale, sweaty and in a lot of discomfort. ECG shows ST elevation of 1mm amplitude in the inferior leads. What is the diagnosis?

Your Answer:E. STEMICorrect Answer:Your answer is correct!

2. After giving him Aspirin 300mg & Clopidogrel 600mg as well as titrated intravenous morphine he becomes bradycardic with a rate of 40bpm. What drug would you administer?

Your Answer:A. AtropineCorrect Answer:Your answer is correct!

3. You are in a small district general hospital. Primary Percutaneous Coronary Intervention (PPCI) is not available at your hospital and the cardiologist at the regional PPCI unit feels the patient is not safe for transfer. The decision is taken to thrombolyse him in the unit. What drug would you administer immediately after thrombolysis?

Your Answer:H. HeparinCorrect Answer:Your answer is correct!

Additional Question Info

Q1   -   E

Acute Coronary Syndrome with ST segment elevation (STEMI) is an acute medical emergency.

Patients usually present with one of the following patterns of symptoms:

(1) Abrupt or worsening of previous angina, with symptoms becoming more frequent, more severe or more prolonged and less responsive to nitro glycerine

(2) Prolonged (> 15 minutes) angina occurring at rest, less than 12 hours.

The ECG may show:

(1) ST elevation in 2 or more adjacent leads (corresponding to a coronary artery territory)

(2) 2mm in leads V1-3 or 1mm in other leads or presumed new onset Left Bundle Branch Block 

Page 59: cardiovascular

Q2   -   A

Atropine increases firing of the sinoatrial node (SA) and conduction through the atrioventricular node (AV) of the heart and opposes the actions of the vagus nerve (main action of the vagus nerve of the parasympathetic system on the heart is to decrease heart rate). For symptomatic bradycardia, the usual dosage is 0.5 to 1.0 mg IV, may repeat every 3 to 5 minutes up to a maximum dose of 3.0 mg.

Q3   -   H

Full therapeutic anticoagulation with either an infusion of unfractionated heparin or low molecular weight heparin (e.g., enoxaparin sodium) is the next step after thromolysis.

Other adjunctive therapy to consider includes intravenous beta-blocker (metoprolol 5 mg IV slow bolus at 0 min, 5 min and 10 min to give a total dose of 15 mg) then oral therapy. IV beta-blockers decreases mortality when given early in acute myocardial infarction (antiarrhythmic and antihypertensive properties as well as ability to reduce ischaemia and they minimize the imbalance between myocardial supply and demand by reducing afterload and wall stress) though the evidence is less clear in the reperfusion therapy setting.

Also ACE-inhibitors when started within 24 hours reduce morbidity and mortality.

 

Send us your feedback about this question.

Extended Matching QuestionMaximum Mark:   3Mark Awarded:   2Time taken to answer:   2 minutes 49 seconds.Main QuestionFor each patient below, choose the most appropriate diagnosis. Select ONE option only from the list above. Each option may be selected once, more than once or not at all.Answers

1. A 30 year old female presents 3 weeks post partum with shortness of breath, haemoptysis and pleuritic chest pain worse on inspiration.

Your Answer:E. Pulmonary embolismCorrect Answer:Your answer is correct!

2. An 18 year old male complains of anterolateral chest wall pain which is tender on palpation after helping his friend move house the previous day.

Your Answer:C. CostochondritisCorrect Answer:Your answer is correct!

3. A 70 year old female presents with crushing retrosternal chest pain which radiates to the jaw. She has had this pain intermittently for the last 3 years and all previous cardiac investigations have been normal.

Your Answer:G. Gastroesophageal reflux diseaseCorrect Answer:A. Oesophageal spasm

Additional Question Info

Q1   -   E

Pulmonary embolism (PE) is a blockage of the pulmonary artery or one of its branches, usually occurring from an embolized dislodged deep vein thrombus usually from the pelvis or legs. Venous

Page 60: cardiovascular

stasis from long haul flights is a risk factor. Other risk factors include recent surgery, stroke or MI; disseminated malignancy; thrombophilia; pregnancy, Pill or HRT.

The classic triad of haemoptysis, dyspnoea and chest pain is rarely the case occurring in fewer than 20% of patients. Signs may include cyanosis, tachypnoea, tachycardia, hypotension, raised JVP or pleural rub. ECG may show tachycardia, AF, RBBB, right ventricular strain, SIQIIITIII rare. ABG may show ↓PaO2 & ↓PaCO2. Do D-dimers only in those who do not have a high probability of PE as a negative result will exclude need for imaging. Positive result does not prove diagnosis and imaging required. CXR may show oligaemia or small pleural effusion. CTPA (or V/Q scan) is investigation of choice. Tx is at least 3mths of warfarin (LMWH until INR>2).

Q2   -   C

Costochondritis is an inflammatory process of the costochondral or costosternal joints that causes localized pain and tenderness. Any of the 7 costochondral junctions may be affected, and more than 1 site is affected in 90% of cases. The second to fifth costochondral junctions most commonly are involved. The pain often comes after a period of strenuous activity and is exacerbated by moving the trunk and deep breathing. While similar, Tietze’s syndrome is not identical to costochondritis. Tietze’s syndrome is characterized by swelling of the costal cartilages, while in costochondritis there is no swelling.

Q3   -   A

Oesophageal spasm is a relatively uncommon condition that more commonly affects elderly women. Oesophageal spasm can be divided into diffuse oesophageal spasm (where there are uncoordinated oesophageal contractions so that several sections of the oesophagus can contract at once and food is unable to pass normally causing dysphagia) and a nutcracker oesophagus (where contractions are coordinated but with an excessive amplitude so that food can usually pass down the oesophagus but pain is common).

Usually, symptoms occur less than once a month. In severe cases, they can occur several times a week or at every meal. Oesophageal spasm can cause episodes of severe, crushing central retrosternal pain. Because the heart and oesophagus are in such close proximity, distinguishing oesophageal from cardiac pain can be difficult and oesophageal spasm is often initially diagnosed as angina pectoris. Oesophageal pain can be gripping, boring, pressing or stabbing. It is usually felt in the anterior chest, throat or epigastrium and can radiate to the neck, back or upper arms, as with cardiac chest pain. Intermittent dysphagia can cause patients to stop eating until symptoms abate. This can take from 30 minutes to several hours. Patients may also complain of globus (the sensation of something stuck in throat/gullet), regurgitation of food and heartburn.

A barium swallow confirms the diagnosis of diffuse oesophageal spasm. There is a characteristic corkscrew oesophagus seen due to multiple simultaneous oesophageal contractions. Nutcracker oesophagus does not have such a characteristic appearance.

Page 61: cardiovascular

It can be managed conservatively by avoiding precipitating factors, e.g. hot or cold food. Muscle relaxants may be effective, e.g. isosorbide mononitrate and nifedipine and Proton pump inhibitors may be needed if there is associated reflux. Surgery (endoscopic balloon dilatation of the gastro-oesophageal sphincter, laparoscopic Heller myotomy is thought to be the surgical treatment of choice for diffuse oesophageal spasm) may be an option if medical measures are unsuccessful.    

 

Send us your feedback about this question.

  Terms & Conditions | Privacy Policy | Site map | Share this page ©2010 MCQBank   Web Design by DotPeak javascript:; javascript:;